LSAT and Law School Admissions Forum

Get expert LSAT preparation and law school admissions advice from PowerScore Test Preparation.

 Administrator
PowerScore Staff
  • PowerScore Staff
  • Posts: 8916
  • Joined: Feb 02, 2011
|
#27200
Complete Question Explanation

Parallel Reasoning—SN. The correct answer choice is (D)

The stimulus features conditional reasoning, which is identifiable by the use of the sufficient condition indicator “if.” The first sentence contains the following relationship:
  • Juan went to party ..... :arrow: ..... Maria enjoy party
The first clause of the second sentence indicates that the necessary condition has not occurred:
  • Maria enjoy party
On the basis of this information, the author concludes that it is unlikely that the sufficient condition occurred:
  • Juan went to party
Note that this argumentation is in the form of a contrapositive, but that there is a slight twist in the language: the phrase “highly unlikely” appears just before the necessary condition in the first sentence, and when this event has been found to occur, the author transfers the “highly unlikely” to the conclusion. This actually makes sense because the “highly unlikely” makes the conditional relationship less than absolute, so when Maria enjoys the party, according to the first sentence then it is possible Juan went to the party, it is just not likely. The entire form of the argument, then, appears as follows:
  • Premise: ..... ..... Juan went to party ..... :arrow: ..... Maria enjoy partyUnlikely

    Premise/conclusion: ..... Maria enjoy party ..... :arrow: ..... Juan went to partyUnlikely
The correct answer choice should contain a contrapositive argument form, and also have the reduced form of the “highly unlikely” or a similar term modifying both the necessary condition and the conclusion.

Answer choice (A): This argument introduces a third term “lose our goalie” and does not follow the form in the stimulus.

Answer choice (B): This argument is a valid form, but it is not in a contrapositive form. This is actually a Repeat form:
  • Premise: ..... ..... Sell insurance ..... :arrow: ..... probably friendly

    Premise/conclusion: ..... Sell insuranceKapinski ..... :arrow: ..... probably friendlyKapinski
Note that the meaning of “probably friendly” and “most people...are friendly” is identical.

Answer choice (C): This answer choice is incorrect because the argument is in a form that is different than that of the stimulus. Without accounting for every difference, the argument is in the basic form of:
  • Premise: ..... Fair ..... :arrow: ..... Likely to win

    Premise: ..... Fair ..... :arrow: ..... Likely to win

    Conclusion: ..... Fair
As this general form does not match the argument in the stimulus, this answer choice is incorrect.

Answer choice (D): This is the correct answer choice. The argument is in a contrapositive form and uses the synonymous term “quite unlikely.” The only difference between this argument and that in the stimulus is that the order of the premises and conclusion differ. However, the order of the pieces of the argument is not an element that must be paralleled, so this answer is correct.

Answer choice (E): This is a tricky answer. The form of this argument is similar to that in the stimulus:
  • Premise: ..... Fair

    Premise: ..... WinPopov ..... :arrow: ..... Fair

    Conclusion: ..... WinPopov
Although the language in the premises uses some probabilistic language (“probably” and “probably will not”), the conclusion does not feature that language. This difference, although minor, is sufficient to eliminate this answer from contention. Remember, all the relevant elements of the argument must be paralleled, not just some of the elements.
 sakfi
  • Posts: 18
  • Joined: Dec 04, 2018
|
#62797
Hi,

Would answer C be a mistaken negation?
And answer choice E is wrong because of the conclusion. It's too definitive while in the stimulus it says "highly unlikely"?

Thank you.
 James Finch
PowerScore Staff
  • PowerScore Staff
  • Posts: 943
  • Joined: Sep 06, 2017
|
#62802
Hi Sakfi,

No, answer choice (C) isn't a Mistaken Negation, but rather contains a logical gap. It gives us two conditional statements as premises, which serve to create a biconditional relationship, but then never gives us that any sufficient condition is actually true. This means that we can't logically conclude anything about the necessary condition, and yet that's exactly what it does. The stimulus gives us a condition as true (Maria enjoyed the party) and uses that to infer the conclusion, so this answer choice cannot parallel the stimulus.

(E) does exactly what you say: as in the stimulus, it gives us a conditional relationship, and the negation of the necessary condition (so the sufficient condition for the contrapositive) as true. Where it goes wrong is jettisoning the probability that ws present in the original conditional and giving its condition 100% certainty, whereas it can only be inferred as likely to happen.

Hope this helps!

Get the most out of your LSAT Prep Plus subscription.

Analyze and track your performance with our Testing and Analytics Package.